Αρχιμήδης 2014-2015

Συντονιστές: cretanman, ΔΗΜΗΤΡΗΣ ΙΩΑΝΝΟΥ, socrates

Άβαταρ μέλους
Demetres
Γενικός Συντονιστής
Δημοσιεύσεις: 8989
Εγγραφή: Δευ Ιαν 19, 2009 5:16 pm
Τοποθεσία: Λεμεσός/Πύλα
Επικοινωνία:

Re: Αρχιμήδης 2014-2015

#41

Μη αναγνωσμένη δημοσίευση από Demetres » Τρί Φεβ 17, 2015 7:09 pm

jasonmaths4ever έγραψε:
Al.Koutsouridis έγραψε:Μιας και ο φάκελος είναι για την προετοιμασία παραθέτω την επίσημη λύση για την Άσκηση 7 σε απόκρυψη σε περίπτωση που κάποιος θέλει να ασχοληθεί ακόμα.
Έστω n=100. Ονομάζουμε μια ακολουθία φυσικών αριθμών ενδιαφέρουσα, αν οποιοιδήποτε δυο διαδοχικοί όροι της διαφέρουν το πολύ κατά 1. Για κάθε ενδιαφέρουσα ακολουθία a_1,α_2, …,a_n θεωρούμε την ακολουθία διαφοράς b_i=a_{i+1} -a_{i} (i=1,2,..,n-1). Κάθε όρος της ακολουθίας διαφοράς είναι ίσος με 0, 1 ή -1. οπότε το πλήθος όλων των ακολουθιών διαφοράς είναι 3^{n-1}.

Θα απαριθμήσουμε αρχικά το πλήθος S όλων των ενδιαφερουσών ακολουθιών, το ελάχιστο στοιχείο των οποίων δεν υπερβαίνει το 3. Εξετάζουμε μια τυχαία ακολουθία διαφοράς b_1,b_2,..,b_{99}. Οποιοισδήποτε δύο ενδιαφέρουσες ακολουθίες που αντιστοιχούν σε αυτήν διαφέρουν στο να αθροίσουμε τον ίδιο αριθμό σε κάθε όρο τους. Άρα μεταξύ αυτών υπάρχει ακριβώς μία ακολουθία με ελάχιστο στοιχείο ίσο με 1, 2 ή 3. Έτσι έχουμε S=3 (3^{n-1}) =3^{n}.

Στο S υπολογίστηκαν όλες οι ακολουθίες που θα μπορούσε να γράψει ο Πέτρος αλλά και κάποιες πλεονάζουσες, αυτές στις οποίες δεν συναντάται το 3. Είναι εύκολο να δει κανείς ότι αν σε μια ενδιαφέρουσα ακολουθία συναντάται αριθμός μικρότερος του 3 και μεγαλύτερος του 3 τότε θα συναντάται και το 3. Όμως το ελάχιστο στοιχείο κάθε πλεονάζουσας ακολουθίας δεν υπερβαίνει το 2 άρα και κάθε όρος της δεν υπερβαίνει το 2. Οπότε όλες οι πλεονάζουσες ακολουθίες αποτελούνται απ τους αρθμους 1 και 2. Από την άλλη κάθε ακολουθία που αποτελείται από 1 και 2 είναι ενδιαφέρουσα και είναι πλεονάζουσα.

Επομένως οι πλεονάζουσες ακολουθίες είναι 2^n τον αριθμό και το ζητούμενο πλήθος των ακολουθιών είναι S-2^n = 3^n -2^n.

Υγ. Η παραπάνω λύση ήταν μια από τις προτυνόμενες λύσεις η δεύτερη ήταν με μαθηματική επαγωγή.
Μπορεί να μην κατάλαβα εγώ καλά, αλλά τι γίνεται αν μια από αυτές τις ακολουθίες έχει έστω και έναν αρνητικό όρο;
Δεν έχουν αρνητικούς όρους. Αυτό που κάνει είναι το εξής (θα το κάνω για n=10 αντί για n=100).

Υπάρχουν 3^9 ακολουθίες διαφοράς b_1,b_2,\ldots,b_9 όπου κάθε b_i ισούται με -1 ή 0 ή 1. Π.χ. μια ακολουθία διαφοράς είναι η 1,0,-1,-1,1,1,0,1,-1. Αυτή η ακολουθία διαφοράς αντιστοιχεί σε πολλές ακολουθίες που την έχουν ως ακολουθία διαφοράς. Π.χ. στην 4,5,5,4,3,4,5,5,6,5 ή στην 0,1,1,0,-1,0,1,1,2,1 κ.τ.λ. Σε όλες αυτές τις ακολουθίες, πάντα ο μικρότερος όρος είναι ο πέμπτος. Υπάρχει λοιπόν μόνο μία ακολουθία που έχει μικρότερο όρο το 1 και έχει ακολουθία διαφοράς την 1,0,-1,-1,1,1,0,1,-1. Αυτή είναι η 2,3,3,2,1,2,3,3,4,3.

Οπότε υπάρχουν ακριβώς 3^9 ακολουθίες που ο μικρότερός τους όρος είναι ο 1. Υπάρχουν άλλες 3^9 με μικρότερο όρο το 2 και άλλες 3^9 με μικρότερο όρο το 3. Συνολικό λοιπόν 3^{10} όπου ο μικρότερος όρος είναι ένας από τους 1,2 ή 3. Από αυτές μόνο οι 2^{10} δεν έχουν το 3 ως όρο. Πιο συγκεκριμένα αυτές που όλοι οι όροι τους ισούνται είτε με 1 είτε με 2. Οπότε τελικά έχουμε 3^{10} - 2^{10} ακολουθίες με τις ιδιότητες που θέλουμε.


Mihalis_Lambrou
Επιμελητής
Δημοσιεύσεις: 15767
Εγγραφή: Κυρ Δεκ 21, 2008 2:04 am

Re: Αρχιμήδης 2014-2015

#42

Μη αναγνωσμένη δημοσίευση από Mihalis_Lambrou » Τετ Φεβ 18, 2015 12:55 am

jasonmaths4ever έγραψε:
nickthegreek έγραψε:Άσκηση 9 (Για Seniors και juniors)

Το άθροισμα ορισμένων φυσικών (όχι κατ' ανάγκη διαφορετικών) αριθμών είναι 100. Πόσο μεγάλο μπορεί να είναι το γινόμενό τους;
... Η μέγιστη λοιπόν τιμή του γινομένου p είναι 2^r = 4^m.
Ιάσονα, για δες το ξανά αυτό. Βρήκες μέγιστο 2^{50} , που προκύπτει από το 2+2+...+2=100. Όμως το
1+3+3+...+3=100 δίνει γινόμενο 3^{33} που είναι μεγαλύτερο:

3^{33}= 3^{26}3^7=9^{13}3^7> 8^{13}3^7=(2^3)^{13}3^7=
=2^{39}\cdot 2187> 2^{39}\cdot 2048=2^{39}2^{11}=2^{50}


socrates
Επιμελητής
Δημοσιεύσεις: 6461
Εγγραφή: Δευ Μαρ 09, 2009 1:47 pm
Τοποθεσία: Θεσσαλονίκη
Επικοινωνία:

Re: Αρχιμήδης 2014-2015

#43

Μη αναγνωσμένη δημοσίευση από socrates » Τετ Φεβ 18, 2015 1:09 am

Άσκηση 14
Να βρεθούν οι ακέραιες λύσεις τις εξίσωσης

\displaystyle{x^4 −2x^3 +x = y^4 + 3y^2 +y.}



Άσκηση 15
Έστω n ένας θετικός ακέραιος. Τοποθετούμε αυθαίρετα στην περιφέρεια ενός κύκλου τους αριθμούς 1, 2, 3,....., 2n. Σε κάθε χορδή που ενώνει δύο από αυτά τα σημεία αντιστοιχούμε έναν αριθμό που είναι ίσος με την απόλυτη τιμή της διαφοράς των αριθμών σε αυτά τα σημεία.
Να δείξετε ότι μπορούμε να επιλέξουμε n μη τεμνόμενες ανά δύο χορδές τέτοιες ώστε το άθροισμα των αριθμών που αντιστοιχούν σε αυτές να ισούται με n^2 .


Θανάσης Κοντογεώργης
Άβαταρ μέλους
Demetres
Γενικός Συντονιστής
Δημοσιεύσεις: 8989
Εγγραφή: Δευ Ιαν 19, 2009 5:16 pm
Τοποθεσία: Λεμεσός/Πύλα
Επικοινωνία:

Re: Αρχιμήδης 2014-2015

#44

Μη αναγνωσμένη δημοσίευση από Demetres » Σάβ Φεβ 21, 2015 10:48 am

Demetres έγραψε:Άσκηση 12 (Για Juniors)

Δυο πιόνια βρίσκονται σε διαφορετικά σημεία του επιπέδου με ακέραιες συντεταγμένες. . Σε κάθε βήμα επιτρέπεται να επιλέξουμε ένα από τα πιόνια και να το τοποθετήσουμε σε ένα άλλο σημείο με ακέραιες συντεταγμένες με την προϋπόθεση ότι η απόσταση μεταξύ των δύο πιονιών να μείνει σταθερή. Μπορούμε μετά από πεπερασμένο αριθμό βημάτων να εναλλάξουμε τις θέσεις των δύο πιονιών;

Επεξεργασία: Βελτίωση εκφώνησης.
Όχι δεν γίνεται. Ας υποθέσουμε ότι γίνεται και ας πάρουμε ένα παράδειγμα όπου η απόσταση μεταξύ των δύο πιονιών είναι η μικρότερη δυνατή. Χωρίς βλάβη της γενικότητας, μπορώ να μεταφέρω τα πιόνια ώστε το A να βρίσκεται στην θέση (0,0) και το B στην θέση (x_B,y_B).

Περίπτωση 1: Οι x_B,y_B είναι και οι δύο περιττοί. Δείξτε ότι όποτε μετακινώ το πιόνι B θα έχει πάντα και τις δύο συντεταγμένες περιττές, ενώ το πιόνι A θα έχει πάντα και τις δύο συντεταγμένες άρτιες. Άρα το B δεν μπορεί ποτέ να καταλήξει στην θέση (0,0), άτοπο.

Περίπτωση 2: Ένα από τα x_B,y_B είναι περιττός και το άλλο είναι άρτιος. Δείξτε ότι όποτε μετακινώ το πιόνι B θα έχει πάντα μια περιττή και μια άρτια συντεταγμένη ενώ το πιόνι A θα έχει πάντα και τις δύο συντεταγμένες άρτιες ή και τις δύο περιττές. Άρα το B δεν μπορεί ποτέ να καταλήξει στην θέση (0,0), άτοπο.

Περίπτωση 3: Οι x_B,y_B είναι και οι δύο άρτιοι. Δείξτε ότι και το A και το B θα έχουν πάντα άρτιες συντεταγμένες. Τότε όμως αν το A ξεκινούσε από το (0,0) και το B από το (x_B/2,y_B/2) πάλι θα είχαμε παράδειγμα με μικρότερη όμως απόσταση μεταξύ των A και B. Άτοπο.


panagiotis99
Δημοσιεύσεις: 133
Εγγραφή: Δευ Φεβ 04, 2013 8:24 pm
Τοποθεσία: Αθηνα

Re: Αρχιμήδης 2014-2015

#45

Μη αναγνωσμένη δημοσίευση από panagiotis99 » Δευ Φεβ 23, 2015 10:12 am

Άσκηση 16 (Juniors)

Να βρεθούν όλοι οι θετικοί ακέραιοι a,b έτσι ώστε a^{5}b+3 , b^{5}a+3 να είναι ταυτόχρονα τέλειοι κύβοι ακεραίων


socrates
Επιμελητής
Δημοσιεύσεις: 6461
Εγγραφή: Δευ Μαρ 09, 2009 1:47 pm
Τοποθεσία: Θεσσαλονίκη
Επικοινωνία:

Re: Αρχιμήδης 2014-2015

#46

Μη αναγνωσμένη δημοσίευση από socrates » Τετ Μαρ 11, 2015 6:12 pm

panagiotis99 έγραψε:Άσκηση 16 (Juniors)

Να βρεθούν όλοι οι θετικοί ακέραιοι a,b έτσι ώστε a^{5}b+3 , b^{5}a+3 να είναι ταυτόχρονα τέλειοι κύβοι ακεραίων
Χρησιμοποιούμε \displaystyle{\pmod 9.}
Δείτε: http://artofproblemsolving.com/communit ... 32p3041819


Θανάσης Κοντογεώργης
gavrilos
Δημοσιεύσεις: 1032
Εγγραφή: Παρ Δεκ 07, 2012 4:11 pm

Re: Αρχιμήδης 2014-2015

#47

Μη αναγνωσμένη δημοσίευση από gavrilos » Τετ Μαρ 11, 2015 9:51 pm

Προτείνω να συνεχίσουμε εν' όψει του προκριματικού.

Κάνω την αρχή και αναμένω ανταπόκριση.

ΑΣΚΗΣΗ 17 (κυρίως για Seniors):

Να βρείτε τον αριθμό των μεταθέσεων \displaystyle{(a_{1},a_{2},\ldots ,a_{2014})} του \displaystyle{(1,2,\ldots ,2014)} για τις οποίες ισχύει \displaystyle{i+a_{i}\leq j+a_{j}} αν ισχύει \displaystyle{1\leq i\leq j\leq 2014}


Γιώργος Γαβριλόπουλος
jason.prod
Δημοσιεύσεις: 141
Εγγραφή: Τρί Φεβ 25, 2014 5:29 pm

Re: Αρχιμήδης 2014-2015

#48

Μη αναγνωσμένη δημοσίευση από jason.prod » Πέμ Μαρ 12, 2015 7:23 pm

gavrilos έγραψε:Προτείνω να συνεχίσουμε εν' όψει του προκριματικού.

Κάνω την αρχή και αναμένω ανταπόκριση.
Και εγώ συμφωνώ να συνεχίσουμε για τον προκριματικό.


Προδρομίδης Κυπριανός-Ιάσων
opener
Δημοσιεύσεις: 15
Εγγραφή: Κυρ Ιούλ 06, 2014 3:30 pm

Re: Αρχιμήδης 2014-2015

#49

Μη αναγνωσμένη δημοσίευση από opener » Παρ Μαρ 13, 2015 1:46 am

gavrilos έγραψε: ΑΣΚΗΣΗ 17 (κυρίως για Seniors):

Να βρείτε τον αριθμό των μεταθέσεων \displaystyle{(a_{1},a_{2},\ldots ,a_{2014})} του \displaystyle{(1,2,\ldots ,2014)} για τις οποίες ισχύει \displaystyle{i+a_{i}\leq j+a_{j}} αν ισχύει \displaystyle{1\leq i\leq j\leq 2014}
Μια προσπάθεια:
Απάντηση:2^{2013}
Λύση:

Λύνουμε το πρόβλημα για τυχαίο n. Έστω s_n το ζητούμενο πλήθος. Πρώτα αποδεικνύουμε το εξής λήμμα:

Λήμμα: i+a_i \leq j+a_j, \forall 1 \leq i \leq j \leq n \Leftrightarrow a_i \leq a_{i+1}+1, \forall i=1,2,...,n-1.
Άπόδειξη: Το \Rightarrow είναι προφανές(j=i+1). Για το \Leftarrow έχουμε επαγωγικά:
a_i \leq a_{i+k}+k, για κάθε k τέτοιο ώστε να ορίζονται οι δείκτες. Έτσι θέτοντας τώρα j=k+i \Leftrightarrow k=j-i έχουμε a_i \leq a_j+j-i, δηλαδή το ζητούμενο.

Έτσι σύμφωνα με το λήμμα αρκεί να μετρήσουμε το πλήθος των μεταθέσεων (a_{1},a_{2},\ldots ,a_{n}) του (1,2,\ldots ,n) για τις οποίες ισχύει a_i \leq a_{i+1}+1, \forall i=1,2,...,n-1. Αυτές τις ονομάζουμε καλές n-άδες. Θα αποδείξουμε επαγωγικά ότι s_n=1+\sum_{i=1}^{n-1}{s_i}, με s_1=1.

Για n=1 ισχύει. Έστω ισχύει έως το n-1. Το δείχνουμε για n τώρα.

Αρχικά θέτουμε a_1=1 και παρατηρούμε ότι κάθε καλή (n-1)-αδα των 1,2,...,n-1 αντιστοιχεί σε μια καλή n-αδα του n και αντίστροφα:
(a_1,...,a_{n-1}) \leftrightarrow (1,a_1+1,...,a_{n-1}+1). Άρα για a_1=1 έχουμε s_{n-1} επιλογές.

Έστω τώρα a_1=2. O a_2 είναι ένας εκ των 1,3,...,n. Άν a_2 \geq 3 αριθμός 1 μετά δεν θα μπορεί να μπει σε καμία θέση. Πράγματι έστω k \in \{2,3,...,n \}:a_k=1. Τότε από το λήμμα 2=a_k +1 \geq a_{k-1} \geq 2 \Rightarrow k=2, άτοπο αφού 1=a_k=a_2>1. Άρα a_2=1. Όμοια με πριν έχουμε s_{n-2} επιλογές λόγω της
(a_1,...,a_{n-2}) \leftrightarrow (2,1,a_1+2,...,a_{n-2}+2).

Αν a_1=3 με παρόμοιο σκεπτικό αποδεικνύουμε ότι a_2=2,a_3=1 και άρα έχουμε άλλες s_{n-3} επιλογές.

Όμοια αν a_1=i έχουμε s_{n-i} επιλογές καλών n-αδων \forall i=1,..,n-1.

Για a_1=n όπως πριν έχουμε a_2=n-1,...,a_i=n+1-i,...,a_n=1, άρα 1 επιλογή.
Επομένως s_n=1+\sum_{i=1}^{n-1}{s_i}, με s_1=1.

Τώρα εύκολα με επαγωγή(ρουτίνα) προκύπτει ότι s_n=2^{n-1} (το οποιο είναι μικρότερο ή ισο απο n!,n \geq 2) και έτσι για n=2014 έχουμε τον ζητούμενο αριθμό.
(Ελπίζω να είναι σωστά!)


Άσκηση 18(για μικρούς):Έστω A=\{ 1,2,...,n+1 \}, B= \{ 1,2,...,n \}.Να βρεθεί το πλήθος των συναρτήσεων f:A \rightarrow B που είναι επί.


jason.prod
Δημοσιεύσεις: 141
Εγγραφή: Τρί Φεβ 25, 2014 5:29 pm

Re: Αρχιμήδης 2014-2015

#50

Μη αναγνωσμένη δημοσίευση από jason.prod » Παρ Μαρ 13, 2015 2:37 pm

opener έγραψε:

Άσκηση 18(για μικρούς):Έστω A=\{ 1,2,...,n+1 \}, B= \{ 1,2,...,n \}.Να βρεθεί το πλήθος των συναρτήσεων f:A \rightarrow B που είναι επί.
Κατ' αρχάς για να είναι επί μία συνάρτηση θα πρέπει για κάθε a \in A να υπάρχει b \in B ώστε f(a) = b. Τα στοιχεία του Α είναι σε πλήθος ένα παραπάνω από τα στοιχεία του Β. Είναι επομένως φανερό ότι για να είναι η δοσμένη συνάρτηση επί και με σύνολα ορισμού και τιμών τα Α και Β αντίστοιχα, θα πρέπει κάθε ένα στοιχείο του Α να αντιστοιχίζεται σε ένα στοιχείο του Β και κάθε στοιχείο του Β να αντιστοιχίζεται σε ένα στοιχείο του Α, εκτός από ένα που θα αντιστοιχίζεται σε 2. Άρα, αν φανταστούμε τα στοιχεία του Β τοποθετημένα σε n θέσεις στη σειρά από το 1 μέχρι το n, τότε τα στοιχεία του Α θα μπορούν να τοποθετηθούν σε αυτές τις θέσεις ως εξής:
Το στοιχείο 1 μπορεί να τοποθετηθεί με n τρόπους
Το στοιχείο 2 μπορεί να τοποθετηθεί με n τρόπους
Το στοιχείο 3 μπορεί να τοποθετηθεί με n-1 τρόπους
κλπ.
Το στοιχείο n μπορεί να τοποθετηθεί με 1 τρόπο.
Άρα σύνολο nn(n-1)...1 = nn! = (n+1)! - n! συναρτήσεις.
Δεν είμαι πολύ σίγουρος οπότε διορθώστε με αν είναι.


Προδρομίδης Κυπριανός-Ιάσων
Άβαταρ μέλους
Demetres
Γενικός Συντονιστής
Δημοσιεύσεις: 8989
Εγγραφή: Δευ Ιαν 19, 2009 5:16 pm
Τοποθεσία: Λεμεσός/Πύλα
Επικοινωνία:

Re: Αρχιμήδης 2014-2015

#51

Μη αναγνωσμένη δημοσίευση από Demetres » Παρ Μαρ 13, 2015 3:07 pm

jasonmaths4ever έγραψε:
opener έγραψε:

Άσκηση 18(για μικρούς):Έστω A=\{ 1,2,...,n+1 \}, B= \{ 1,2,...,n \}.Να βρεθεί το πλήθος των συναρτήσεων f:A \rightarrow B που είναι επί.
Κατ' αρχάς για να είναι επί μία συνάρτηση θα πρέπει για κάθε a \in A να υπάρχει b \in B ώστε f(a) = b. Τα στοιχεία του Α είναι σε πλήθος ένα παραπάνω από τα στοιχεία του Β. Είναι επομένως φανερό ότι για να είναι η δοσμένη συνάρτηση επί και με σύνολα ορισμού και τιμών τα Α και Β αντίστοιχα, θα πρέπει κάθε ένα στοιχείο του Α να αντιστοιχίζεται σε ένα στοιχείο του Β και κάθε στοιχείο του Β να αντιστοιχίζεται σε ένα στοιχείο του Α, εκτός από ένα που θα αντιστοιχίζεται σε 2. Άρα, αν φανταστούμε τα στοιχεία του Β τοποθετημένα σε n θέσεις στη σειρά από το 1 μέχρι το n, τότε τα στοιχεία του Α θα μπορούν να τοποθετηθούν σε αυτές τις θέσεις ως εξής:
Το στοιχείο 1 μπορεί να τοποθετηθεί με n τρόπους
Το στοιχείο 2 μπορεί να τοποθετηθεί με n τρόπους
Το στοιχείο 3 μπορεί να τοποθετηθεί με n-1 τρόπους
κλπ.
Το στοιχείο n μπορεί να τοποθετηθεί με 1 τρόπο.
Άρα σύνολο nn(n-1)...1 = nn! = (n+1)! - n! συναρτήσεις.
Δεν είμαι πολύ σίγουρος οπότε διορθώστε με αν είναι.
Η πρώτη γραμμή είναι λανθασμένη, ίσως εκ παραδρομής μιας και η εξήγηση αργότερα είναι σωστή. Η πρώτη γραμμή έπρεπε να λέει ότι για κάθε b υπάρχει a με f(a) = b.

Αργότερα, υπάρχει λάθος στο εξής σημείο:

Έστω π.χ. n=5. Έχουμε πέντε τρόπους να τοποθετήσουμε το 1 και πέντε τρόπους να τοποθετήσουμε το 2. Για το 3 και μετά όμως εξαρτάται από το τι τοποθετήσεις έχουμε ήδη κάνει. Π.χ. αν τοποθετήσουμε και το 1 και το 2 στην ίδια θέση τότε έχουμε τέσσερις επιλογές για το 3. Αν όμως τοποθετήσουμε τα 1 και 2 σε διαφορετικές θέσεις τότε έχουμε πέντε επιλογές για το 3. Επειδή σε διαφορετικές περιπτώσεις έχουμε διαφορετικό αριθμό επιλογών, δεν μπορούμε να χρησιμοποιήσουμε την πολλαπλασιαστική αρχή με αυτόν τον τρόπο.


jason.prod
Δημοσιεύσεις: 141
Εγγραφή: Τρί Φεβ 25, 2014 5:29 pm

Re: Αρχιμήδης 2014-2015

#52

Μη αναγνωσμένη δημοσίευση από jason.prod » Παρ Μαρ 13, 2015 3:14 pm

Ωχ! Πράγματι, έχετε δίκιο. Θα το ξαναδώ πιο προσεκτικά και θα διορθώσω τη λύση.


Προδρομίδης Κυπριανός-Ιάσων
Άβαταρ μέλους
Demetres
Γενικός Συντονιστής
Δημοσιεύσεις: 8989
Εγγραφή: Δευ Ιαν 19, 2009 5:16 pm
Τοποθεσία: Λεμεσός/Πύλα
Επικοινωνία:

Re: Αρχιμήδης 2014-2015

#53

Μη αναγνωσμένη δημοσίευση από Demetres » Παρ Μαρ 13, 2015 3:26 pm

opener έγραψε:
gavrilos έγραψε: ΑΣΚΗΣΗ 17 (κυρίως για Seniors):

Να βρείτε τον αριθμό των μεταθέσεων \displaystyle{(a_{1},a_{2},\ldots ,a_{2014})} του \displaystyle{(1,2,\ldots ,2014)} για τις οποίες ισχύει \displaystyle{i+a_{i}\leq j+a_{j}} αν ισχύει \displaystyle{1\leq i\leq j\leq 2014}
Μια προσπάθεια:
Απάντηση:2^{2013}
Λύση:

(Ελπίζω να είναι σωστά!)
Σωστά! Ίσως να χρειάζεται λίγη περισσότερη εξήγηση. Θα διατύπωνα το πιο κάτω ως ξεχωριστό ισχυρισμό τον οποίο θα αποδείκνυα με επαγωγή.

Αν a_k = 1 τότε για 1 \leqslant i \leqslant k-1 είναι a_{k-i} = i+1.

Η επαγωγή είναι αρκετά απλή και ίσως γι' αυτό να μην την έκανες.


Άβαταρ μέλους
Demetres
Γενικός Συντονιστής
Δημοσιεύσεις: 8989
Εγγραφή: Δευ Ιαν 19, 2009 5:16 pm
Τοποθεσία: Λεμεσός/Πύλα
Επικοινωνία:

Re: Αρχιμήδης 2014-2015

#54

Μη αναγνωσμένη δημοσίευση από Demetres » Παρ Μαρ 13, 2015 4:00 pm

Ας δούμε και μια αριθμοθεωρία:

Άσκηση 19

Ας βρεθούν όλα τα ζεύγη (p,q) πρώτων αριθμών ώστε p^2|q^3+1 και q^2|p^6-1.

Απολογούμαι αν την έχουμε ξαναδεί.


socrates
Επιμελητής
Δημοσιεύσεις: 6461
Εγγραφή: Δευ Μαρ 09, 2009 1:47 pm
Τοποθεσία: Θεσσαλονίκη
Επικοινωνία:

Re: Αρχιμήδης 2014-2015

#55

Μη αναγνωσμένη δημοσίευση από socrates » Σάβ Μαρ 14, 2015 3:21 am

Άσκηση 20(για μικρούς)
Έστω a_1, a_2, ... , a_{99} θετικοί ακέραιοι μικρότεροι του 100.
Αν το άθροισμα οποιονδήποτε δύο ή περισσότερων από αυτούς δε διαιρείται με το 100, να δείξετε ότι είναι όλοι ίσοι.

Πηγή: https://www.dropbox.com/sh/w9mfy9qtjs68 ... 2.pdf?dl=0


Θανάσης Κοντογεώργης
jason.prod
Δημοσιεύσεις: 141
Εγγραφή: Τρί Φεβ 25, 2014 5:29 pm

Re: Αρχιμήδης 2014-2015

#56

Μη αναγνωσμένη δημοσίευση από jason.prod » Σάβ Μαρ 14, 2015 12:14 pm

jasonmaths4ever έγραψε:Ωχ! Πράγματι, έχετε δίκιο. Θα το ξαναδώ πιο προσεκτικά και θα διορθώσω τη λύση.
Για πάμε άλλη μία προσπάθεια. Η λύση μου βέβαια δεν είναι πολύ κομψή, διότι την παρουσιάζω σε μορφή αθροίσματος, το οποίο δεν έχω μετατρέψει σε τηλεσκοπικό. Συνεχίζω από το σημείο όπου υπήρχε το λάθος. Αν το k-οστό στοιχείο μπει σε πιασμένη θέση, 2 \le k \le n+1 τότε έχω συνολικά n^k (n-k)! συναρτήσεις, και έτσι αυτές είναι στο σύνολο \sum_{k=2}^{n+1} n^k (n-k)! συναρτήσεις.
Σας παρακαλώ όποιος μπορεί να μετατρέψει το παραπάνω άθροισμα σε τηλεσκοπικό και αν χρειαστεί να με διορθώσει.


Προδρομίδης Κυπριανός-Ιάσων
Άβαταρ μέλους
Αρχιμήδης 6
Δημοσιεύσεις: 1205
Εγγραφή: Παρ Αύγ 27, 2010 11:27 pm
Τοποθεσία: ΚΑΛΑΜΑΤΑ

Re: Αρχιμήδης 2014-2015

#57

Μη αναγνωσμένη δημοσίευση από Αρχιμήδης 6 » Σάβ Μαρ 14, 2015 3:58 pm

Άσκηση 21
Να λυθεί στους φυσικούς η εξίσωση,

x^3+x^2+x=y^2+y


Λάθε βιώσας-Επίκουρος
Κανακάρης Δημήτριος.
Άβαταρ μέλους
Αρχιμήδης 6
Δημοσιεύσεις: 1205
Εγγραφή: Παρ Αύγ 27, 2010 11:27 pm
Τοποθεσία: ΚΑΛΑΜΑΤΑ

Re: Αρχιμήδης 2014-2015

#58

Μη αναγνωσμένη δημοσίευση από Αρχιμήδης 6 » Κυρ Μαρ 15, 2015 12:36 pm

Demetres έγραψε:Ας δούμε και μια αριθμοθεωρία:

Άσκηση 19

Ας βρεθούν όλα τα ζεύγη (p,q) πρώτων αριθμών ώστε p^2|q^3+1 και q^2|p^6-1.

Απολογούμαι αν την έχουμε ξαναδεί.
Καταρχήν ελέγχω τις περιπτώσεις αν p=2 , p=3 , q=2 , q=3 συνεπώς τα ζεύγη (p,q)=(2,3),(3,2) αποτελούν λύση οπότε πλέον θα ερευνήσω την εξίσωση για p,q>3

p^2|q^3+1=(q+1)(q^2-q+1) Επειδή p>3 τότε ο p^2 θα διαιρεί ακριβώς έναν παράγοντα .

Άρα είτε p^2|q+1 είτε p^2|q^2-q+1

Ότι και να ισχύει το μόνο σίγουρο είναι ότι p^2\leq{q^2-q+1} (1)

Επίσης ισχύει ότι q^2|p^6-1=(p-1)(p+1)(p^2-p+1)(p^2+p+1)

Επειδή όμως είναι αδύνατον ο p να διαιρεί 2 διαφορετικούς παράγοντες τότε ο p^2 θα διαιρεί ακριβώς έναν άρα
είτε q^2|p+1 είτε q^2|p-1 είτε q^2|p^2-p+1 είτε q^2|p^2+p+1 . Σε κάθε περίπτωση θα ισχύει ότι

q^2\leq{p^2+p+1} (2)

Ας προσθέσω τις (1),(2) και τελικά...

p^2+q^2\leq{p^2+q^2+p-q+2}

q\leq{p+2} (3) αλλά από (1) q>p άρα

q=p+2 η q=p+1 αντικαθιστώντας δεν θα έχω λύση για p,q>3 . Άρα μοναδικές λύσεις τα ζεύγη

(p,q)=(2,3),(3,2)

Y.Γ Ωραία διαιρετότητα αλλά όχι ιδιαίτερα σφιχτή αφού ουσιαστικά μπορώ εύκολα να κατεβάσω τα άνω φράγματα των p^2,q^2 αν και δεν χρειάστηκε .


Λάθε βιώσας-Επίκουρος
Κανακάρης Δημήτριος.
vzf
Δημοσιεύσεις: 310
Εγγραφή: Κυρ Φεβ 28, 2010 11:11 pm

Re: Αρχιμήδης 2014-2015

#59

Μη αναγνωσμένη δημοσίευση από vzf » Κυρ Μαρ 15, 2015 6:59 pm

ΑΣΚΗΣΗ 22
Αποδείξτε ότι η εξίσωση y^2=x^3+23 δεν έχει ακέραιες λύσεις.


Άβαταρ μέλους
Αρχιμήδης 6
Δημοσιεύσεις: 1205
Εγγραφή: Παρ Αύγ 27, 2010 11:27 pm
Τοποθεσία: ΚΑΛΑΜΑΤΑ

Re: Αρχιμήδης 2014-2015

#60

Μη αναγνωσμένη δημοσίευση από Αρχιμήδης 6 » Κυρ Μαρ 15, 2015 11:58 pm

vzf έγραψε:ΑΣΚΗΣΗ 22
Αποδείξτε ότι η εξίσωση y^2=x^3+23 δεν έχει ακέραιες λύσεις.
Λήμμα .

Αν p|x^2+y^2 , x,y θετικοί ακέραιοι και p=3mod4 τότε p|x και p|y

Απόδειξη

p|x^2+y^2|x^{2*\dfrac{p-1}{2}}+y^{2*\dfrac{p-1}{2}} (γιατί \dfrac{p-1}{2} περιττός)

p|x^{p-1}+y^{p-1}

Αν (p,x)=(p,y)=1 τότε από θεώρημα euler θα πρέπει p|2 αδύνατον άρα p|x και p|y


y^2=x^3+23

Γενικά βλέπω ότι y^2=0,1mod4 άρα x^3=1,2mod4 άρα

x=1mod4 και y=0mod2

(****Βλέπε σχόλια στο τέλος***) Ψάχνω κατάλληλους a,b ώστε y^2+a^2=x^3+b^3 (1) και δεδομένου ότι y^2=x^3+23 τότε αφαιρώντας θα ισχύει ότι

a^2=b^3-23 με προφανή λύση την (a,b)=(2,3) άρα πάω στην (1) συνεπώς

y^2+2^2=x^3+3^3

y^2+2^2=(x+3)(x^2-3x+9) (2)

Απέδειξα παραπάνω ότι x=1mod4 άρα x^2-3x+9=3mod4

Ο αριθμός λοιπόν x^2-3x+9 είναι της μορφής 4k+3 άρα θα έχει πρώτο διαιρέτη της μορφής αυτής αφού αν δεν είχε τότε ως περιττός θα είχε μόνο της μορφής 4k+1 που το γινόμενο τους δεν θα ήταν ποτέ της μορφής 4k+3 .

Έστω p ο πρώτος διαιρέτης του x^2-3x+9 που είναι της μορφής 4k+3 άρα από (2) έχω

p|x^2-3x+9|y^2+2^2 αλλά σύμφωνα με το λήμμα επειδή p=3mod4 και p|y^2+2^2 τότε p|2 άρα p=2 κάτι που είναι αδύνατον .


****Η εύρεση των a,b στην περίπτωση μας ήταν απλή αλλά σε άλλες εξισώσεις με μεγαλύτερες σταθερές αντί 23 η εύρεση των κατάλληλων a,b καθίσταται κουραστική και έτσι έχουμε μια "απλοποίηση" της εύρεσης τους με τον τρόπο που παρέθεσα.(Αναγωγή σε μια άλλη διοφαντική!)
Μάλιστα σε κάποια βιβλία υπάρχει έρευνα για το πότε μια τέτοια εξίσωση είναι αδύνατη ανάλογα με την σχέση των a,b.


Λάθε βιώσας-Επίκουρος
Κανακάρης Δημήτριος.
Απάντηση

Επιστροφή σε “Θέματα διαγωνισμών (ΕΜΕ, ΚΥΜΕ, BMO, JBMO, IMO, Kangaroo κλπ)”

Μέλη σε σύνδεση

Μέλη σε αυτήν τη Δ. Συζήτηση: Δεν υπάρχουν εγγεγραμμένα μέλη και 33 επισκέπτες